有名問題・定理から学ぶ数学

Well-Known Problems and Theorems in Mathematics

数式を枠からはみ出さずに表示するためには, 画面を横に傾けてください.

関数

関数

定義《関数》

  • 数の集合 $A$ の要素 $x$ に応じて数の集合 $B$ の $1$ つの要素 $f(x)$ を対応させる規則を, $A$ から $B$ への関数 (function) と呼び, \[ f:A\to B;x\mapsto f(x)\] や $f:A\to B$, または単に $f(x)$ や $f$ で表す. また, $A$ を $f$ の定義域 (domain) と呼び, $B$ を $f$ の終域または行き先 (target) と呼ぶ.
  • 関数 $f:A\to B$ について, $A$ の各要素 $x$ に対して $f(x)$ を $f$ による $x$ の (value) と呼ぶ. $f$ による値全体からなる $B$ の部分集合を $f$ の値域 (range) と呼ぶ.
  • 数の集合 $A$ に対して, $\mathrm{id}_A(x) = x$ で定まる関数 $\mathrm{id}_A:A\to A$ を $A$ 上の恒等関数 (identity function) と呼ぶ.

定義《恒等式, 方程式》

 $f:A\to B,$ $g:A\to B$ を定義域と終域が等しい関数とする.
(1)
$f,$ $g$ が $A$ のすべての要素 $x$ に対して $f(x) = g(x)$ を満たすとき, $f$ と $g$ は等しいといい, $f = g$ と表す. このような等式を恒等式 (identity) と呼ぶ.
(2)
恒等式であるとは限らない等式 $f(x) = g(x)$ を方程式 (equation) と呼び, $f(a) = g(a)$ を満たす $A$ の要素 $a$ を方程式 $f(x) = g(x)$ の (solution) と呼ぶ. $f(x) = g(x)$ のすべての解を求めることを方程式 $f(x) = g(x)$ を解く (solve) という.

問題《集合の特性関数》

 $U$ を数の集合とする. $U$ の各部分集合 $A$ に対して, $U$ を定義域とする関数 $\chi _A(x)$ を \[\chi _A = \begin{cases} 1 & (x \in A), \\ 0 & (x \notin A) \end{cases}\] で定める.
(A)
$U$ の部分集合 $A,$ $B$ に対して,
(1)
$\chi _{A\cup B}(x) = \chi _A(x)+\chi _B(x)-\chi _{A\cap B}(x) \quad \cdots [1]$ 
(2)
$\chi _{A\cap B}(x) = \chi _A(x)\chi _B(x) \quad \cdots [2]$ 
(3)
$\chi _{\bar A}(x) = 1-\chi _A(x) \quad \cdots [3]$ 
(4)
$\chi _{\bar A\cap\bar B}(x) = 1-\chi _{A}(x)-\chi _{A}(x)+\chi _{A\cap B}(x) \quad \cdots [4]$ 
が成り立つことを示せ.
(B)
$U$ が $n$ 個の要素 $x_1,\cdots,x_n$ からなる有限集合であるとき, $U$ の部分集合 $A$ の要素の個数は \[ n(A) = \chi _A(x_1)+\cdots +\chi _A(x_n)\] であることを示せ.
標準定理$2022/11/15$$2022/12/26$

解答例

(A)
(1)
(i)
$x \in A,$ $x \in B$ のとき. $x \in A\cup B,$ $x \in A\cap B$ から, \[\begin{aligned} \chi _{A\cup B}(x) &= 1, \\ \chi _A(x)+\chi _B(x)-\chi _{A\cap B}(x) &= 1+1-1 = 1 \end{aligned}\] の値は等しい.
(ii)
$x \in A,$ $x \notin B$ のとき. $x \in A\cup B,$ $x \notin A\cap B$ から, \[\begin{aligned} \chi _{A\cup B}(x) &= 1, \\ \chi _A(x)+\chi _B(x)-\chi _{A\cap B}(x) &= 1+0-0 = 1 \end{aligned}\] の値は等しい.
(iii)
$x \notin A,$ $x \in B$ のとき. $x \in A\cup B,$ $x \notin A\cap B$ から, \[\begin{aligned} \chi _{A\cup B}(x) &= 1, \\ \chi _A(x)+\chi _B(x)-\chi _{A\cap B}(x) &= 0+1-0 = 1 \end{aligned}\] の値は等しい.
(iv)
$x \notin A,$ $x \notin B$ のとき. $x \notin A\cup B,$ $x \notin A\cap B$ から, \[\begin{aligned} \chi _{A\cup B}(x) &= 0, \\ \chi _A(x)+\chi _B(x)-\chi _{A\cap B}(x) &= 0+0-0 = 0 \end{aligned}\] の値は等しい.
(i)~(iv) から, $[1]$ が成り立つ.
(2)
$\chi _{A\cap B}(x),$ $\chi _A(x)\chi _B(x)$ は $0,$ $1$ の値しかとらないことに注意する. $U$ の要素 $x$ に対して \[\begin{aligned} \chi _{A\cap B}(x) = 1 &\iff x \in A\cap B \\ &\iff x \in A,\ x \in B \\ &\iff \chi _A(x) = 1,\ \chi _B(x) = 1 \\ &\iff \chi _A(x)\chi _B(x) = 1 \end{aligned}\] であるから, $[2]$ が成り立つ.
(3)
$\chi _{\bar A}(x),$ $1-\chi _A(x)$ は $0,$ $1$ の値しかとらないことに注意する. $U$ の要素 $x$ に対して \[\begin{aligned} \chi _{\bar A}(x) = 1 &\iff x \in \bar A \\ &\iff x \notin A \\ &\iff \chi _A(x) = 0 \\ &\iff 1-\chi _A(x) = 1 \end{aligned}\] であるから, $[3]$ が成り立つ.
(4)
$[2],$ $[3]$ により, \[\begin{aligned} \chi _{\bar A\cap\bar B}(x) &= \chi _{\bar A}(x)\chi _{\bar B}(x) \quad (\because [2]) \\ &= \{ 1-\chi _A(x)\}\{ 1-\chi _B(x)\} \quad (\because [3]) \\ &= 1-\chi _A(x)-\chi _B(x)+\chi _A(x)\chi _B(x) \\ &= 1-\chi _A(x)-\chi _B(x)+\chi _{A\cap B}(x) \quad (\because [2]) \end{aligned}\] が成り立つ.
(B)
$n(A) = m,$ $A = \{ x_{i_1},\cdots,x_{i_m}\},$ $\bar A = \{ x_{i_{m+1}},\cdots,x_{i_n}\}$ のとき, $\chi _A(x)$ の定義により \[\begin{aligned} \chi _A(x_{i_1}) = \cdots = \chi _A(x_{i_m}) &= 1, \\ \chi _A(x_{i_{m+1}}) = \cdots = \chi _A(x_{i_n}) &= 0 \end{aligned}\] であるから, \[\begin{aligned} &\chi _A(x_1)+\cdots +\chi _A(x_n) \\ &= \chi _A(x_{i_1})+\cdots +\chi _A(x_{i_m})+\chi _A(x_{i_{m+1}})+\cdots +\chi _A(x_{i_n}) \\ &= m\cdot 1+(n-m)\cdot 0 \\ &= m = n(A) \end{aligned}\] が成り立つ.

別解

(A)
(4)
ド・モルガンの法則, $[3],$ $[1]$ により, \[\begin{aligned} \chi _{\bar A\cap\bar B}(x) &= \chi _{\overline{A\cup B}}(x) \\ &= 1-\chi _{A\cup B}(x) \quad (\because [3]) \\ &= 1-\{\chi _A(x)+\chi _B(x)-\chi _{A\cap B}(x)\} \quad (\because [1]) \\ &= 1-\chi _A(x)-\chi _B(x)+\chi _{A\cap B}(x) \end{aligned}\] が成り立つ.

参考

  • 一般に, 空間 $U$ の部分集合 $A$ に対して, 同様に $A$ の「特性関数」(characteristic function) と呼ばれる関数 $\chi _A(x)$ が定義される. この関数は,「ベクトル解析」「ルベーグ積分論」「測度論」「確率論」などの解析学の諸理論で基本的な役割を果たす.
  • $[4]$ は, 次の「包除原理」(principle of inclusion and exclusion) に一般化される: 集合 $U$ の部分集合 $A_1,$ $\cdots,$ $A_r$ に対して, \[\begin{aligned} &\chi _{\overline{A_1}\cap\cdots\cap\overline{A_r}}(x) \\ &= 1-\sum_{1 \leqq i_1 \leqq r}\chi _{A_{i_1}}(x)+\sum_{1 \leqq i_1 < i_2 \leqq r}\chi _{A_{i_1}\cap A_{i_2}}(x) \\ &\qquad -\sum_{1 \leqq i_1 < i_2 < i_3 \leqq r}\chi _{A_{i_1}\cap A_{i_2}\cap A_{i_3}}(x)+\cdots +(-1)^r\chi _{A_1\cap\cdots\cap A_r}(x) \\ &= \sum_{k = 0}^r(-1)^k\sum_{1 \leqq i_1 < \cdots < i_k \leqq r}\chi _{A_{i_1}\cap\cdots\cap A_{i_k}}(x) \end{aligned}\] が成り立つ. ここで, $\displaystyle\sum_{1 \leqq i_1 < \cdots < i_k \leqq r}$ は $1 \leqq i_1 < \cdots < i_k \leqq r$ を満たすすべての整数の組 $(i_1,\cdots,i_k)$ にわたる和を表す.
  • それぞれをもとの位置と異なる位置に並べ替える順列は「完全順列」または「攪乱順列」(derangement) と呼ばれる. $n$ 個のものの「完全順列」の総数 $a_n$ は「モンモール数」(Montmort number) と呼ばれ, $a_n$ を求める問題は「モンモールの問題」と呼ばれる. $1,$ $\cdots,$ $n$ の順列において $i$ 番目に $i$ を並べる順列全体の集合 $A_i$ $(1 \leqq i \leqq r)$ に「包除原理」を適用すると, \[ a_n = \sum\limits_{k = 0}^n(-1)^k{}_n\mathrm C_k\,(n-k)! = n!\sum\limits_{k = 0}^n\frac{(-1)^k}{k!}\] が得られる. この公式については, こちらこちらも参照.
  • 正の整数 $n$ に, $n$ と互いに素な $1$ 以上 $n$ 以下の整数の個数を対応させる関数 $\varphi (n)$ を,「オイラーの (トーシェント) 関数」(Euler's (totient) function) と呼ぶ. $n$ の素因数が $p_1,$ $\cdots,$ $p_r$ であるとき, $U = \{ 1,\cdots,n\}$ を全体集合, $p_i$ の倍数全体の集合を $A_i$ $(1 \leqq i \leqq r)$ として,「包除原理」 \[\chi _{\overline{A_1}\cap\cdots\cap\overline{A_r}}(x) = \sum_{k = 0}^r(-1)^k\sum_{1 \leqq i_1 < \cdots < i_k \leqq r}\chi _{A_{i_1}\cap\cdots\cap A_{i_k}}(x)\] の等式に $x = 1,$ $\cdots,$ $n$ を代入し, 辺々を加えると, (B) により \[\begin{aligned} \varphi (n) &= \sum_{k = 0}^r(-1)^k\sum_{1 \leqq i_1 < \cdots < i_k \leqq r}\frac{n}{p_{i_1}\cdots p_{i_k}} \\ &= n\sum_{k = 0}^r(-1)^k\sum_{1 \leqq i_1 < \cdots < i_k \leqq r}\frac{1}{p_{i_1}\cdots p_{i_k}} \\ &= n\left( 1-\frac{1}{p_1}\right)\cdots\left( 1-\frac{1}{p_r}\right) \quad \cdots [\ast ] \end{aligned}\] が得られる. ここで, $\varphi (n)$ が $\overline{A_1}\cap\cdots\cap\overline{A_r}$ の要素の個数に等しいこと, $A_{i_1}\cap\cdots\cap A_{i_k}$ の要素の個数は $1$ 以上 $n$ 以下の $p_{i_1}\cdots p_{i_k}$ の倍数の個数に等しく $\dfrac{n}{p_{i_1}\cdots p_{i_k}}$ であることを用いた. さらに, $[\ast ]$ から, 互いに素な正の整数 $m,$ $n$ に対して \[\varphi (mn) = \varphi (m)\varphi (n)\] が成り立つという「オイラーの関数」の「乗法性」(multiplicity) が導かれる.

問題《関数の偶奇分解》

 共通の定義域 $I$ をもつ実数値関数 $f(x),$ $g(x)$ に対して, $f(x),$ $g(x)$ の和, 差をそれぞれ \[ (f+g)(x) = f(x)+g(x), \quad (f-g)(x) = f(x)-g(x)\] で定める.
 実数全体の部分集合 $I$ が原点に関して対称であるとき, $I$ を定義域とする関数 $f(x),$ $g(x)$ に対して, 次のことを示せ.
(1)
関数 \[ f_0(x) = \frac{f(x)+f(-x)}{2}, \quad f_1(x) = \frac{f(x)-f(-x)}{2}\] はそれぞれ偶関数, 奇関数である.
(2)
$f(x),$ $g(x)$ がともに偶関数ならば, $(f+g)(x),$ $(f-g)(x)$ は偶関数である.
(3)
$f(x),$ $g(x)$ がともに奇関数ならば, $(f+g)(x),$ $(f-g)(x)$ は奇関数である.
(4)
$f(x)$ が偶関数かつ奇関数であるならば, $f(x) = 0$ である.
(5)
$f(x)$ は偶関数と奇関数の和としてただ $1$ 通りに表される.
標準定理$2023/06/20$$2023/08/05$

解答例

(1)
\[\begin{aligned} f_0(-x) &= \frac{f(-x)+f(x)}{2} = f_0(x), \\ f_1(-x) &= \frac{f(-x)-f(x)}{2} = -f_1(x) \end{aligned}\] が成り立つから, $f_0(x),$ $f_1(x)$ はそれぞれ偶関数, 奇関数である.
(2)
$f(x),$ $g(x)$ がともに偶関数ならば, \[\begin{aligned} (f+g)(-x) &= f(-x)+g(-x) \\ &= f(x)+g(x) \\ &= (f+g)(x), \\ (f-g)(-x) &= f(-x)-g(-x) \\ &= f(x)-g(x) \\ &= (f-g)(x) \end{aligned}\] が成り立つから, $(f+g)(x),$ $(f-g)(x)$ は偶関数である.
(3)
$f(x),$ $g(x)$ がともに奇関数ならば, \[\begin{aligned} (f+g)(-x) &= f(-x)+g(-x) \\ &= \{ -f(x)\} +\{ -g(x)\} = -\{ f(x)+g(x)\} \\ &= -(f+g)(x), \\ (f-g)(-x) &= f(-x)-g(-x) \\ &= \{ -f(x)\} -\{ -g(x)\} = -\{ f(x)-g(x)\} \\ &= -(f-g)(x) \end{aligned}\] が成り立つから, $(f+g)(x),$ $(f-g)(x)$ は奇関数である.
(4)
$f(x)$ が偶関数かつ奇関数であるとき, 定義域において \[ f(x) = f(-x) = -f(x)\] よって $2f(x) = 0$ つまり $f(x) = 0$ を満たす.
(5)
関数 $f(x)$ は偶関数 $f_0(x),$ 奇関数 $f_1(x)$ を用いて \[ f(x) = \frac{f(x)+f(-x)}{2}+\frac{f(x)-f(-x)}{2} = f_0(x)+f_1(x)\] と表される. さらに, $f(x)$ が偶関数 $g_0(x),$ 奇関数 $g_1(x)$ を用いて \[ f(x) = g_0(x)+g_1(x)\] と表されたとする. このとき, \[ f_0(x)+f_1(x) = g_0(x)+g_1(x)\] から \[ f_0(x)-g_0(x) = g_1(x)-f_1(x)\] となる. (2) により左辺は偶関数の差で偶関数, (3) により右辺は奇関数の差で奇関数であることから, (4) により \[ f_0(x)-g_0(x) = g_1(x)-f_1(x) = 0\] つまり \[ f_0(x) = g_0(x), \quad f_1(x) = g_1(x)\] となる. ゆえに, $f(x)$ は偶関数と奇関数の和としてただ $1$ 通りに表される.

参考

  • 常に $0$ でない関数 $f(x),$ $g(x)$ の一方が偶関数, 他方が奇関数であるとき, $(f+g)(x)$ は偶関数でも奇関数でもない.
  • 同様の議論により, すべての $2$ 変数関数 $f(x,y)$ は 「対称式」と「交代式」の和として \[ f(x,y) = \frac{f(x,y)+f(y,x)}{2}+\frac{f(x,y)-f(y,x)}{2}\] とただ $1$ 通りに表されることが証明できる.

問題《偶関数・奇関数の積・商の偶奇性》

 共通の定義域 $I$ をもつ実数値関数 $f(x),$ $g(x)$ に対して, $f(x),$ $g(x)$ の積, 商をそれぞれ \[ (fg)(x) = f(x)g(x), \quad \left(\frac{f}{g}\right) (x) = \frac{f(x)}{g(x)} \quad (g(x) \neq 0)\] で定める.
 実数全体の部分集合 $I$ が原点に関して対称であるとき, $I$ を定義域とする関数 $f(x),$ $g(x)$ に対して, 次のことを示せ.
(A)
$f(x),$ $g(x)$ がともに偶関数, またはともに奇関数ならば, $(fg)(x),$ $\displaystyle\left(\frac{f}{g}\right) (x)$ は偶関数である.
(B)
$f(x),$ $g(x)$ の一方が偶関数, 他方が奇関数ならば, $(fg)(x),$ $\displaystyle\left(\frac{f}{g}\right) (x)$ は奇関数である.
基本定理$2023/06/20$$2023/06/22$

解答例

(A)
  • $f(x),$ $g(x)$ がともに偶関数ならば, \[\begin{aligned} (fg)(-x) &= f(-x)g(-x) \\ &= f(x)g(x) \\ &= (fg)(x), \\ \left(\frac{f}{g}\right) (-x) &= \frac{f(-x)}{g(-x)} \\ &= \frac{f(x)}{g(x)} \\ &= \left(\frac{f}{g}\right) (x) \end{aligned}\] が成り立つから, $(fg)(x),$ $\displaystyle\left(\frac{f}{g}\right) (x)$ は偶関数である.
  • $f(x),$ $g(x)$ がともに奇関数ならば, \[\begin{aligned} (fg)(-x) &= f(-x)g(-x) \\ &= \{ -f(x)\}\{ -g(x)\} = f(x)g(x) \\ &= (fg)(x), \\ \left(\frac{f}{g}\right) (-x) &= \frac{f(-x)}{g(-x)} \\ &= \frac{-f(x)}{-g(x)} = \frac{f(x)}{g(x)} \\ &= \left(\frac{f}{g}\right) (x) \end{aligned}\] が成り立つから, $(fg)(x),$ $\displaystyle\left(\frac{f}{g}\right) (x)$ は偶関数である.
(B)
$(fg)(x) = (gf)(x)$ であるから, $f(x)$ が偶関数, $g(x)$ が奇関数であるとしても一般性を失わない. このとき, \[\begin{aligned} (fg)(-x) &= f(-x)g(-x) \\ &= f(x)\{ -g(x)\} = -f(x)g(x) \\ &= -(fg)(x), \\ \left(\frac{f}{g}\right) (-x) &= \frac{f(-x)}{g(-x)} \\ &= \frac{f(x)}{-g(x)} = -\frac{f(x)}{g(x)} \\ &= -\left(\frac{f}{g}\right) (x) \end{aligned}\] が成り立つから, $(fg)(x),$ $\displaystyle\left(\frac{f}{g}\right) (x)$ は奇関数である.

合成関数・逆関数

定義《合成関数》

 関数 $f(x)$ の値域が関数 $g(x)$ の定義域に含まれるとき, $x$ に $g(f(x))$ を対応させることで定まる関数を $f(x)$ と $g(x)$ の合成関数 (composite function) と呼び, $(g\circ f)(x)$ で表す.

定義《逆関数》

 関数 $f(x)$ の値域に含まれる任意の値 $y$ に対して $y = f(x)$ なる $x$ がただ $1$ つ定まるとき, $y$ に $x$ を対応させることで定まる関数を $f(x)$ の逆関数 (inverse function) と呼ぶ.

定理《合成関数の性質》

 $f:A\to B,$ $g:B\to C,$ $h:C\to D$ を関数とする.
(1)
$h\circ (g\circ f) = (h\circ g)\circ f$ が成り立つ.
(2)
$f\circ\mathrm{id}_A = \mathrm{id}_B\circ f = f$ が成り立つ.
(3)
$f$ の逆関数が存在するならば, それはただ $1$ つに定まる (それを $f^{-1}$ で表す).

証明

(1)
合成関数の定義により, $A$ のすべての要素 $x$ に対して \[\begin{aligned} (h\circ (g\circ f))(x) &= h((g\circ f)(x)) = h(g(f(x)) \\ ((h\circ g)\circ f)(x) &= (h\circ g)(f(x)) = h(g(f(x)) \end{aligned}\] が成り立つから, $h\circ (g\circ f) = (h\circ g)\circ f$ である.
(2)
合成関数の定義と $\mathrm{id}_A$ の定義により, $A$ のすべての要素 $x$ に対して \[\begin{aligned} (f\circ\mathrm{id}_A)(x) = f(\mathrm{id}_A(x)) &= f(x), \\ (\mathrm{id}_B\circ f)(x) = \mathrm{id}_B(f(x)) &= f(x) \end{aligned}\] が成り立つから, $f\circ\mathrm{id}_A = \mathrm{id}_B\circ f = f$ である.
(3)
$g$ と $g'$ が $f$ の逆写像であるとすると \[ g \!=\! g\circ\mathrm{id}_B \!=\! g\circ (f\circ g') \!=\! (g\circ f)\circ g' \!=\! \mathrm{id}_A\circ g' \!=\! g'\] となるから, $f$ の逆写像は高々 $1$ つしか存在しない.

問題《$n$ 文字の置換の性質》

 $1$ 以上 $n$ 以下の整数全体 $A$ を定義域とする関数 $f(x)$ の値域が $A$ と一致するとする. $k$ 個の $f(x)$ を合成して得られる関数を $f^k(x)$ で表すとき, $f^k(1)$ $(1 \leqq k \leqq n)$ が互いに異なるとする.
(1)
関数として $f^n(x) = x$ が成り立つことを示せ.
(2)
$f(x)$ の逆関数を, $f^k(x)$ $(1 \leqq k \leqq n)$ の形に表せ.
標準定理$2019/08/03$$2022/06/11$

解答例

(1)
仮定から $\{ f^1(1),\cdots,f^n(1)\} = \{ 1,\cdots,n\}$ であるので, $f^k(1) = 1$ なる整数 $k$ が $1 \leqq k \leqq n$ の範囲に存在する.
 仮に $1 \leqq k \leqq n-1$ であるとすると, \[ f^{k+1}(1) = (f\circ f^k)(1) = f(f^k(1)) = f(1),\] $2 \leqq k+1 \leqq n$ となり, $f^k(1)$ $(1 \leqq k \leqq n)$ が互いに異なることに反する.
 よって, $f^n(1) = 1$ が成り立つ. $\{ 1,\cdots,n\} = \{ f^1(1),\cdots,f^n(1)\}$ であるから, $1 \leqq x \leqq n$ の範囲にある各整数 $x$ に対して, $x = f^k(1)$ なる整数 $k$ が $1 \leqq k \leqq n$ の範囲に存在して, \[ f^n(x) = f^n(f^k(1)) = f^k(f^n(1)) = f^k(1) = x\] が成り立つ.
(2)
$y = f(x)$ とおくと, (1) の結果により \[ f^{n-1}(y) = f^{n-1}(f(x)) = f^n(x) = x\] となるから, $f(x)$ の逆関数は $f^{n-1}(x)$ である.

参考

  • 次の条件を満たす演算 $*$ の定義された集合 $G\,(\neq \varnothing )$ を「群」(group) と呼ぶ.
    (G0)
    $G$ のすべての要素 $a,$ $b$ に対して, $a*b \in G$ が成り立つ.
    (G1)
    $(a*b)*c = a*(b*c)$ $(a,b,c \in G)$ が成り立つ.
    (G2)
    $e*a = a*e = a$ $(a \in G)$ なる $G$ の要素 $e$ が存在する.
    (G3)
    $G$ の各要素 $a$ に対して, $a*a^{-1} = a^{ -1}*a = e$ なる $G$ の要素 $a^{-1}$ が存在する.
    (II) の $e$ を $G$ の「単位元」(identity element), (III) の $a^{-1}$ を $a$ の「逆元」(inverse element) と呼ぶ.
  • 定義域と値域が同一の集合 $A$ であり, 逆関数をもつ関数を $A$ 上の「置換」(permutation) と呼ぶ. 集合 $A$ 上の「置換」全体の集合 $G$ は合成を演算として「群」をなす. これを $A$ 上の「置換群」(permutation group) と呼ぶ. 特に, 集合 $\{ 1,\cdots,n\}$ 上の「置換群」を「$n$ 次対称群」(symmetric group of degree $n$) と呼ぶ. その「単位元」は $e(x) = x$ $(x \in A)$ である.
  • 「群」$G$ の要素 $g$ について, $k$ 個の $g$ に演算を施した結果が $G$ の「単位元」になるような正の整数 $k$ の最小値を $g$ の「位数」(order) と呼ぶ.
  • $a_1,$ $\cdots,$ $a_r$ が集合 $A$ の相異なる要素で, $a_{r+1} = a_1$ であるとき, $f(a_k) = a_{k+1}$ $(1 \leqq k \leqq r),$ $f(x) = x$ $(x \neq a_1,\ \cdots,\ a_r)$ を満たす $A$ 上の「置換」$f(x)$ を長さ $r$ の「巡回置換」(cyclic permutation) と呼ぶ. 長さ $r$ の「巡回置換」の「位数」は $r$ である.

問題《コラッツ予想に関する整数の性質》

 正の整数全体を定義域とする関数 $f(x)$ を
$x$ が偶数のとき $f(x) = \dfrac{x}{2},$ $x$ が奇数のとき $f(x) = 3x+1$
により定め, $f(x)$ を $r$ 回合成した関数 $f^r(x)$ を \[ f^1(x) = f(x), \quad f^r(x) = (f\circ f^{r-1})(x) \quad (r > 1)\] により定める. 正の整数 $n$ が条件
ある正の整数 $r$ に対して $f^r(n) = 1$ $\cdots [\ast ]$
を満たすかどうかを考える. 非負整数 $k$ に対して, 次のことを示せ.
(A)
$n = 4k+1$ が $[\ast ]$ を満たすためには, $n = 3k+1$ が $[\ast ]$ を満たせば十分である.
(B)
$n = 4k+3$ が $[\ast ]$ を満たすためには, $n = 9k+8$ が $[\ast ]$ を満たすことが必要である.
標準先例$2023/10/12$$2022/10/17$

解答例

(A)
$n = 3k+1$ が $[\ast ]$ を満たすとする. このとき, \[ f^s(3k+1) = 1\] を満たす正の整数 $s$ が存在する. また, \[\begin{aligned} f(4k+1) &= 3(4k+1)+1 = 12k+4, \\ f^2(4k+1) &= f(12k+4) = \frac{12k+4}{2} = 6k+2, \\ f^3(4k+1) &= f(6k+2) = \frac{6k+2}{2} = 3k+1 \end{aligned}\] であるから, \[ f^{s+3}(4k+1) = f^s(f^3(4k+1)) = f^s(3k+1) = 1\] が成り立つ. よって, $n = 4k+1$ は $r = s+3$ について $[\ast ]$ を満たす. ゆえに, $n = 4k+1$ が $[\ast ]$ を満たすためには, $n = 3k+1$ が $[\ast ]$ を満たせば十分である.
(B)
$n = 4k+3$ が $[\ast ]$ を満たすとする. このとき, \[ f^s(4k+3) = 1\] を満たす正の整数 $s$ が存在する. \[\begin{aligned} f(4k+3) &= 3(4k+3)+1 = 12k+10 \neq 1, \\ f^2(4k+3) &= f(12k+10) = \frac{12k+10}{2} = 6k+5 \neq 1, \\ f^3(4k+3) &= f(6k+5) = 3(6k+5)+1 = 18k+16 \neq 1, \\ f^4(4k+3) &= f(18k+16) = \frac{18k+16}{2} = 9k+8 \neq 1 \end{aligned}\] であるから, $s \geqq 5$ であって \[ 1 = f^s(4k+3) = f^{s-4}(f^4(4k+3)) = f^{s-4}(9k+8)\] が成り立つ. よって, $n = 9k+8$ は $r = s-4$ について $[\ast ]$ を満たす. ゆえに, $n = 4k+3$ が $[\ast ]$ を満たすためには, $n = 9k+8$ が $[\ast ]$ を満たすことが必要である.

参考

  • すべての正の整数 $n$ が $[\ast ]$ を満たすという予想は「コラッツ予想」(Collatz conjecture) と呼ばれ, 初期値が $2^{68} \fallingdotseq 2.95\times 10^{20}$ 以下のとき正しいことがコンピューターにより確かめられている ($2020$ 年現在).
  • $2$ で割れるだけ割るという操作により, 偶数 $n = 2^ek$ ($e$: 正の整数, $k$: 奇数) が $[\ast ]$ を満たすためには, 奇数 $n = k$ が $[\ast ]$ を満たせば十分である.
  • 初期値が $2k-1$ 以下 ($k$: 正の整数) のとき予想が正しいとすると, この事実により初期値が $n = 2k$ のときも予想は正しいが, $n = 2k+1$ が $[\ast ]$ を満たすかどうかはわからない. このとき, \[ f(4k+2) = 2k+1 > 2k-1\] であるから, 偶数 $n = 4k+2$ $$ が $[\ast ]$ を満たすかどうかもわからない.
  • 初期値が $4k-1$ 以下 ($k$: 正の整数) のとき予想が正しいとすると, 上記の事実と (A) により $n = 4k,$ $4k+1,$ $4k+2$ のときも予想は正しいが, $n = 4k+3$ が $[\ast ]$ を満たすかどうかはわからない. このとき, $k$ を $3$ で割った余りが $1$ であるならば, \[ f^3\left(\dfrac{16k+11}{3}\right) = 4k+3 > 4k-1\] であるから, $4$ で割った余りが $1$ である奇数 $n = \dfrac{16k+11}{3}$ が $[\ast ]$ を満たすかどうかもわからない.

問題《$1$ 次分数関数の逆関数》

 $a,$ $b,$ $c,$ $d$ を $c \neq 0,$ $ad \neq bc$ なる実数とする.
(1)
関数 $f(x) = \dfrac{ax+b}{cx+d}$ の逆関数 $f^{-1}(x)$ を求めよ.
(2)
$f(x) = f^{-1}(x)$ となるための必要十分条件を求めよ.
標準定理$2018/08/09$$2018/08/09$

解答例

(1)
$y = \dfrac{ax+b}{cx+d}$ とおく. 分母を払って変形すると \[\begin{aligned} (cx+d)y &= ax+b, \\ (cy-a)x &= -dy+b \end{aligned}\] となるから, \[ cy-a = \frac{c(ax+b)-a(cx+d)}{cx+d} = \frac{bc-ad}{cx+d} \neq 0\] に注意すると, \[ x = \frac{-dy+b}{cy-a} = \frac{dy-b}{-cy+a}\] となる. ゆえに, $f(x)$ の逆関数は \[ f^{-1}(x) = \frac{dx-b}{-cx+a}\] である.
(2)
$f(x) = f^{-1}(x)$ が成り立つとする. このとき, \[\frac{ax+b}{cx+d} = \frac{dx-b}{-cx+a}\] であるから, $x \neq -\dfrac{d}{c},$ $\dfrac{a}{c}$ なるすべての実数 $x$ に対して \[ (ax+b)(-cx+a) = (cx+d)(dx-b)\] つまり \[ -acx^2+(a^2-bc)x+ab = cdx^2+(d^2-bc)x-bd\] が成り立つ. 両辺の係数を比較すると \[ -ac = cd, \quad a^2-bc = d^2-bc, \quad ab = -bd\] つまり \[ c(a+d) = 0,\ (a+d)(a-d) = 0,\ b(a+d) = 0\] となるから, $c \neq 0$ に注意すると, 求める条件が $a+d = 0$ であるとわかる.

問題《マチンの公式》

(1)
$\tan\alpha = \dfrac{1}{5}$ とする. $\tan 2\alpha,$ $\tan 4\alpha$ の値を求めよ.
(2)
関数 $\tan x\ \left( -\dfrac{\pi}{2} < x < \dfrac{\pi}{2}\right)$ の逆関数を $\arctan x$ で表す. \[\frac{\pi}{4} = 4\arctan\frac{1}{5}-\arctan\frac{1}{239} \quad \cdots [\ast ]\] が成り立つことを示せ.
標準定理$2015/09/21$$2022/06/11$

解答例

(1)
$2$ 倍角の公式により, \[\begin{aligned} &\tan 2\alpha = \frac{2\tan\alpha}{1-\tan ^2\alpha} \\ &= \frac{2\cdot\dfrac{1}{5}}{1-\dfrac{1}{25}} = \frac{10}{25-1} = \frac{5}{12} \end{aligned}\] である. よって, 再び $2$ 倍角の公式を使うと, \[\begin{aligned} &\tan 4\alpha = \frac{2\tan 2\alpha}{1-\tan ^22\alpha} \\ &= \frac{2\cdot\dfrac{5}{12}}{1-\dfrac{25}{144}} = \frac{120}{144-25} = \frac{120}{119} \end{aligned}\] が得られる.
(2)
\[\frac{\pi}{4} = 4\alpha -\left( 4\alpha -\frac{\pi}{4}\right), \quad \alpha = \arctan\dfrac{1}{5}\] であるから, \[ 4\alpha -\frac{\pi}{4} = \arctan\frac{1}{239} \quad \cdots [1]\] を示せばよい. 加法定理により, \[\begin{aligned} &\tan\left( 4\alpha -\frac{\pi}{4}\right) = \frac{\tan 4\alpha-\tan\dfrac{\pi}{4}}{1+\tan 4\alpha\tan\dfrac{\pi}{4}} \\ &= \frac{\dfrac{120}{119}-1}{1+\dfrac{120}{119}\cdot 1} = \frac{120-119}{119+120} = \frac{1}{239} \end{aligned}\] が成り立つ. これは $[1]$ の成立を意味するから, $[\ast ]$ が得られた.

参考

  • 三角関数の定義域を適当に制限すれば, その逆関数を定めることができる. 三角関数の逆関数は, 逆数を値にとる関数との混同を避けるため, $\mathrm{arc}$ をつけて表すことが多い.
  • イギリスの天文学者マチンは,「マチンの公式」(Machin's formula) と呼ばれる等式 $[*]$ を「グレゴリー=ライプニッツ級数」 \[\arctan x = \sum\limits_{n = 0}^\infty\frac{(-1)^n}{2n+1}x^{2n+1} \quad (-1 \leqq x \leqq 1)\] (こちらを参照) と結びつけることで, 非常に速く $\pi$ に収束する無限級数 \[\pi = 4\sum\limits_{n = 0}^\infty\frac{(-1)^n}{(2n+1)5^{2n+1}}-4\sum\limits_{n = 0}^\infty\frac{(-1)^n}{(2n+1)239^{2n+1}}\] を発見した.
  • マチンは $[\ast ]$ の他にも, \[\begin{aligned} \frac{\pi}{4} &= \mathrm{arctan}\frac{1}{2}+\mathrm{arctan}\frac{1}{3} \\ &= 2\,\mathrm{arctan}\frac{1}{2}-\mathrm{arctan}\frac{1}{7} \\ &= 2\,\mathrm{arctan}\frac{1}{3}+\mathrm{arctan}\frac{1}{7} \end{aligned}\] などの公式を発見している. シュテルマーにより, \[\pi = a_1\,\mathrm{arctan}\frac{1}{x_1}+a_2\,\mathrm{arctan}\frac{1}{x_2}\] ($a_1,$ $a_2$: 正の整数, $x_1,$ $x_2$: 相異なる $0$ でない整数)の形の公式はこれらの $4$ つのみであることが示されている. 類似の公式については, こちらを参照されたい.

問題《双曲線関数の逆関数》

 $a > 0,$ $a \neq 1$ とする.
(A)
関数 $y = \dfrac{a^x+a^{-x}}{2}$ $(x \geqq 0)$ の逆関数を求めよ.
(B)
関数 $y = \dfrac{a^x-a^{-x}}{2}$ の逆関数を求めよ.
(C)
関数 $y = \dfrac{a^x-a^{-x}}{a^x+a^{-x}}$ の逆関数を求めよ.
標準定理$2020/01/25$$2022/07/27$

解答例

(A)
$y = \dfrac{a^x+a^{-x}}{2}$ から \[\begin{aligned} &2y = a^x+a^{-x} \\ &a^x-2y+a^{-x} = 0 \\ &a^{2x}-2ya^x+1 = 0 \end{aligned}\] であるので, $2$ 次方程式の解の公式により $a^x = y\pm\sqrt{y^2-1}$ が得られるが, $a^x \geqq 1$ $(x \geqq 0),$ $y \geqq 1$ であるから \[ a^x = y+\sqrt{y^2-1}\] である. よって, \[ y = \frac{a^x+a^{-x}}{2} \iff x = \log _a(y+\sqrt{y^2-1})\] であるから, 求める逆関数は \[ y = \log _a(x+\sqrt{x^2-1}) \quad (x \geqq 1)\] である.
(B)
$y = \dfrac{a^x-a^{-x}}{2}$ から \[\begin{aligned} &2y = a^x-a^{-x} \\ &a^x-2y-a^{-x} = 0 \\ &a^{2x}-2ya^x-1 = 0 \end{aligned}\] であるので, $2$ 次方程式の解の公式により $a^x = y\pm\sqrt{y^2+1}$ が得られるが, $a^x \geqq 1$ $(x \geqq 0),$ $y > \sqrt{y^2+1}$ であるから \[ a^x = y+\sqrt{y^2+1}\] である. よって, \[ y = \frac{a^x-a^{-x}}{2} \iff x = \log _a(y+\sqrt{y^2+1})\] であるから, 求める逆関数は \[ y = \log _a(x+\sqrt{x^2+1})\] である.
(C)
$y = \dfrac{a^x-a^{-x}}{a^x+a^{-x}},$ $-1 < y < 1$ から, \[\begin{aligned} (a^x+a^{-x})y &= a^x-a^{-x} \\ (a^{2x}+1)y &= a^{2x}-1 \\ (1-y)a^{2x} &= 1+y \\ a^{2x} &= \frac{1+y}{1-y} \\ 2x &= \log _a\frac{1+y}{1-y} \end{aligned}\] である. よって, \[ y = \frac{a^x-a^{-x}}{a^x+a^{-x}} \iff x = \frac{1}{2}\log _a\frac{1+y}{1-y}\] であるから, 求める逆関数は \[ y = \frac{1}{2}\log _a\frac{1+x}{1-x} \quad (-1 < x < 1)\] である.
問題一覧 (関数と極限)分数関数 無理関数 関数
数列の極限 無限級数
関数の極限 関数の連続性